Chapter 13 Statistics and Probability
Multiple Choice Questions (MCQs)
1 In the formula $\bar{{}x}=a+\dfrac{\Sigma f_i d_i}{\Sigma f_i}$, for finding the mean of grouped data $d_i$ ’s are deviation from $\boldsymbol{{}a}$ of
(a) lower limits of the classes $\quad$ (b) upper limits of the classes
(c) mid-points of the classes $\quad$ (d) frequencies of the class marks
Solution (c) We know that, $d_i=x_i-a$ i.e., $d_i$ ’s are the deviation from a of mid-points of the classes.Show Answer
(a) evenly distributed over all the classes
(b) centred at the class marks of the classes
(c) centred at the upper limits of the classes
(d) centred at the lower limits of the classes
Solution (b) In computing the mean of grouped data, the frequencies are centred at the class marks of the classes.Show Answer
(a) 0 $\quad$ (b) -1 $\quad$ (c) 1 $\quad$ (d) 2
Solution (a) $\because$ $
\bar{{}x}=\dfrac{\sum f_i x_i}{n}
$ $
\begin{aligned}
\Sigma(f_i x_i-\bar{{}x}) = \Sigma f_i x_i-\Sigma \bar{{}x} \\
& =n \bar{{}x}-n \bar{{}x} \\
& =0
\end{aligned}
$Show Answer
(a) $\dfrac{x_i+a}{h}$ $\quad$ (b) $h(x_i-a)$
(c) $\dfrac{x_i-a}{h}$ $\quad$ (d) $\dfrac{a-x_i}{h}$
Solution (c) Given, $\bar{{}x}=a+h \dfrac{\Sigma f_i u_i}{\Sigma f_i}$ Above formula is a step deviation formula. $
u_i=\dfrac{x_i-a}{h}
$Show Answer
(a) mean $\quad$ (b) median $\quad$ (c) mode $\quad$ (d) All of these
Solution (b) Since, the intersection point of less than ogive and more than ogive gives the median on the abscissa.Show Answer
Class | $0-5$ | $5-10$ | $10-15$ | $15-20$ | $20-25$ |
---|---|---|---|---|---|
Frequency | 10 | 15 | 12 | 20 | 9 |
the sum of lower limits of the median class and modal class is
(a) 15 $\quad$ (b) 25 $\quad$ (c) 30 $\quad$ (d) 35
Solution (b) Here, Now, $\dfrac{N}{2}=\dfrac{66}{2}=33$, which lies in the interval $10-15$. Therefore, lower limit of the median class is 10 . The highest frequency is 20, which lies in the interval 15-20. Therefore, lower limit of modal class is 15 . Hence, required sum is $10+15=25$.Show Answer
Class
Frequency
Cumulative
frequency
$0-5$
10
10
$5-10$
15
25
$10-15$
12
37
$15-20$
20
57
$20-25$
9
66
Class | $0-5$ | $6-11$ | $12-17$ | $18-23$ | $24-29$ |
---|---|---|---|---|---|
Frequency | 13 | 10 | 15 | 8 | 11 |
The upper limit of the median class is
(a) 17 $\quad$ (b) 17.5 $\quad$ (c) 18 $\quad$ (d) 18.5
Solution (b) Given, classes are not continuous, so we make continuous by subtracting 0.5 from lower limit and adding 0.5 to upper limit of each class. Here, $\dfrac{N}{2}=\dfrac{57}{2}=28.5$, which lies in the interval 11.5-17.5. Hence, the upper limit is 17.5 .Show Answer
Class
Frequency
Cumulative
frequency
$-0.5-5.5$
13
13
$5.5-11.5$
10
23
$11.5-17.5$
15
38
$17.5-23.5$
8
46
$23.5-29.5$
11
57
Marks | Number of students |
---|---|
Below 10 | 3 |
Below 20 | 12 |
Below 30 | 27 |
Below 40 | 57 |
Below 50 | 75 |
Below 60 | 80 |
the modal class is
(a) $10-20$ $\quad$ (b) 20-30 $\quad$ (c) $30-40$ $\quad$ (d) $50-60$
Solution (c) Here, we see that the highest frequency is 30 , which lies in the interval $30-40$.Show Answer
Marks
Number of students
Cumulative frequency
Below 10
$3=3$
3
$10-20$
$(12-3)=9$
12
$20-30$
$(27-12)=15$
27
$30-40$
$(57-27)=30$
57
$40-50$
$(75-57)=18$
75
$50-60$
$(80-75)=5$
80
Class | $65-85$ | $85-105$ | $105-125$ | $125-145$ | $145-165$ | $165-185$ | $185-205$ |
---|---|---|---|---|---|---|---|
Frequency | 4 | 5 | 13 | 20 | 14 | 7 | 4 |
The difference of the upper limit of the median class and the lower limit of the modal class is
(a) 0 $\quad$ (b) 19 $\quad$ (c) 20 $\quad$ (d) 38
Solution (c) Here, $\dfrac{N}{2}=\dfrac{67}{2}=33.5$ which lies in the interval $125-145$. Hence, upper limit of median class is 145 . Here, we see that the highest frequency is 20 which lies in 125-145. Hence, the lower limit of modal class is 125 . $\therefore$ Required difference $=$ Upper limit of median class -Lower limit of modal class $=145-125=20$Show Answer
Class
Frequency
Cumulative frequency
$65-85$
4
4
$85-105$
5
9
$105-125$
13
22
$125-145$
20
42
$145-165$
14
56
$165-185$
7
63
$185-205$
4
67
Class | $13.8-14$ | $14-14.2$ | $14.2-14.4$ | $14.4-14.6$ | $14.6-14.8$ | $14.8-15$ |
---|---|---|---|---|---|---|
Frequency | 2 | 4 | 5 | 71 | 48 | 20 |
The number of atheletes who completed the race in less than $14.6 s$ is
(a) 11 $\quad$ (b) 71 $\quad$ (c) 82 $\quad$ (d) 130
Solution (c) The number of atheletes who completed the race in less than 14.6 $=2+4+5+71=82$Show Answer
Marks obtained | Number of students |
---|---|
More than or equal to 0 | 63 |
More than or equal to 10 | 58 |
More than or equal to 20 | 55 |
More than or equal to 30 | 51 |
More than or equal to 40 | 48 |
More than or equal to 50 | 42 |
the frequency of the class $30-40$ is
(a) 3 $\quad$ (b) 4 $\quad$ (c) 48 $\quad$ (d) 51
Solution (a) Hence, frequency in the class interval $30-40$ is 3 .Show Answer
Marks obtained
Number of students
$0-10$
$(63-58)=5$
$10-20$
$(58-55)=3$
$20-30$
$(55-51)=4$
$30-40$
$(51-48)=3$
$40-50$
$(48-42)=6$
$50 .$.
$42=42$
(a) 1 $\quad$ (b) $\dfrac{3}{4}$ $\quad$ (c) $\dfrac{1}{2}$ $\quad$ (d) 0
Solution (d) The event which cannot occur is said to be impossible event and probability of impossible event is zero.Show Answer
(a) $\dfrac{1}{3}$ $\quad$ (b) 0.1 $\quad$ (c) 3 $\quad$ (d) $\dfrac{17}{16}$
Solution (d) Since, probability of an event always lies between 0 and 1 .Show Answer
(a) 0.0001 $\quad$ (b) 0.001 $\quad$ (c) 0.01 $\quad$ (d) 0.1
Solution (a) The probability of an event which is very unlikely to happen is closest to zero and from the given options 0.0001 is closest to zero.Show Answer
(a) $P-1$ $\quad$ (b) $P$ $\quad$ (c) $1-P$ $\quad$ (d) $1-\dfrac{1}{P}$
Solution (c) Since, probability of an event + probability of its complementry event $=1$ So, probability of its complementry event $=1$ - Probability of an event $=1-P$Show Answer
(a) less than 100 $\quad$ (b) less than 0 $\quad$ (c) greater than 1 $\quad$ (d) anything but a whole number
Solution (b) We know that, the probability expressed as a percentage always lie between 0 and 100. So, it cannot be less than 0 .Show Answer
(a) $P(A)<0$ $\quad$ (b) $P(A)>1$ $\quad$ (c) $0 \leq P(A) \leq 1$ $\quad$ (d) $-1 \leq P(A) \leq 1$
Solution (c) Since, probability of an event always lies between 0 and 1 .Show Answer
(a) $\dfrac{3}{26}$ $\quad$ (b) $\dfrac{3}{13}$ $\quad$ (c) $\dfrac{2}{13}$ $\quad$ (d) $\dfrac{1}{2}$
Solution (a) In a deck of 52 cards, there are 12 face cards i.e., 6 red and 6 black cards. So, probability of getting a red face card $=\dfrac{6}{52}=\dfrac{3}{26}$Show Answer
(a) $\dfrac{1}{7}$ $\quad$ (b) $\dfrac{2}{7}$ $\quad$ (c) $\dfrac{3}{7}$ $\quad$ (d) $\dfrac{5}{7}$
Solution (a) A non-leap year has 365 days and therefore 52 weeks and 1 day. This 1 day may be Sunday or Monday or Tuesday or Wednesday or Thursday or Friday or Saturday. Thus, out of 7 possibilities, 1 favourable event is the event that the one day is Sunday. $\therefore$ Required probability $=\dfrac{1}{7}$Show Answer
(a) $\dfrac{1}{6}$ $\quad$ (b) $\dfrac{1}{3}$ $\quad$ (c) $\dfrac{1}{2}$ $\quad$ (d) 0
Solution (a) When a die is thrown, then total number of outcomes $=6$ Odd number less than 3 is 1 only. Number of possible outcomes $=1$ $\therefore \quad$ Required probability $=\dfrac{1}{6}$.Show Answer
(a) 4 $\quad$ (b) 13 $\quad$ (c) 48 $\quad$ (d) 51
Solution (d) In a deck of 52 cards, there are 13 cards of heart and 1 is ace of heart. Hence, the number of outcomes favourable to $E=51$Show Answer
(a) 7 $\quad$ (b) 14 $\quad$ (c) 21 $\quad$ (d) 28
Solution (b) Here, total number of eggs $=400$ Probability of getting a bad egg $=0.035$ $
\begin{matrix}
\Rightarrow & \dfrac{\text{ Number of bad eggs }}{\text{ Total number of eggs }}=0.035 \\
\Rightarrow & \dfrac{\text{ Number of bad eggs }}{400}=0.035 \\
\therefore & \text{ Number of bad eggs }=0.035 \times 400=14
\end{matrix}
$Show Answer
(a) 40 $\quad$ (b) 240 $\quad$ (c) 480 $\quad$ (d) 750
Solution (c) Given, total number of sold tickets $=6000$ Let she bought $x$ tickets. Then, probability of her winning the first prize $=\dfrac{x}{6000}=0.08$ [given] $\Rightarrow \quad x=0.08 \times 6000$ $\therefore \quad x=480$ Hence, she bought 480 tickets.Show Answer
(a) $\dfrac{1}{5}$ $\quad$ (b) $\dfrac{3}{5}$ $\quad$ (c) $\dfrac{4}{5}$ $\quad$ (d) $\dfrac{1}{3}$
Solution (a) Number of total outcomes $=40$ Multiples of 5 between 1 to $40=5,10,15,20,25,30,35,40$ $\therefore$ Total number of possible outcomes $=8$ $\therefore \quad$ Required probability $=\dfrac{8}{40}=\dfrac{1}{5}$Show Answer
(a) $\dfrac{1}{5}$ $\quad$ (b) $\dfrac{6}{25}$ $\quad$ (c) $\dfrac{1}{4}$ $\quad$ (d) $\dfrac{13}{50}$
Solution (c) Total numbers of outcomes $=100$ So, the prime numbers between 1 to 100 are $2,3,5,7,11,13,17,19,23,29,31,37,41$, $43,47,53,56,61,67,71,73,79,83,89$ and 97 . $\therefore$ Total number of possible outcomes $=25$ $\therefore \quad$ Required probability $=\dfrac{25}{100}=\dfrac{1}{4}$Show Answer
(a) $\dfrac{4}{23}$ $\quad$ (b) $\dfrac{6}{23}$ $\quad$ (c) $\dfrac{8}{23}$ $\quad$ (d) $\dfrac{17}{23}$
Show Answer
Solution
(b) Total number of students $=23$
Number of students in house $A, B$ and $C=4+8+5=17$
$\therefore \quad$ Remains students $=23-17=6$
So, probability that the selected student is not from $A, B$ and $C=\dfrac{6}{23}$
Very Short Answer Type Questions
1 The median of an ungrouped data and the median calculated when the same data is grouped are always the same. Do you think that this is a correct statement? Give reason.
Solution Not always, because for calculating median of a grouped data, the formula used is based on the assumption that the observations in the classes are uniformal distributed (or equally spaced).Show Answer
$ \bar{{}x}=a+\dfrac{\sum f_i d_i}{\sum f_i} $
Where, $\boldsymbol{{}a}$ is the assumed mean, $\boldsymbol{{}a}$ must be one of the mid-point of the classes. Is the last statement correct? Justify your answer.
Solution No, it is not necessary that assumed mean consider as the mid-point of the class interval. It is considered as any value which is easy to simplify it.Show Answer
Solution No, the value of these three measures can be the same, it depends on the type of data.Show Answer
Solution Not always, It depends on the given data.Show Answer
Solution No, the probability of each is not $\dfrac{1}{4}$ because the probability of no girl in three children is zero and probability of three girls in three children is one. Justification So, these events are not equally likely as outcome one girl, means $g b b, b g b, b b g$ ’three girls’ means ’ $g 9 g$ ’ and so on.Show Answer
Solution No, the outcomes are not equally likely, because 3 contains half part of the total region, so it is more likely than 1 and 2 , since 1 and 2 , each contains half part of the remaining part of the region.Show Answer
Solution Apoorv throws two dice once. So total number of outcomes $=36$ Number of outcomes for getting product $36=1(6 \times 6)$ $\therefore \quad$ Probability for Apoorv $=\dfrac{1}{36}$ Also, Peehu throws one die, So, total number of outcomes $=6$ Number of outcomes for getting square $36=1(6^{2}=36)$ $\therefore \quad$ Probability for Peehu $=\dfrac{1}{6}=\dfrac{6}{36}$ Hence, Peehu has better chance of getting the number 36 .Show Answer
Solution Yes, probability of each outcome is $\dfrac{1}{2}$ because head and tail both are equally likely events.Show Answer
Solution No, this is not correct. Suppose we throw a die, then total number of outcomes $=6$ Possible outcomes $=1$ or 2 or 3 or 4 or 5 or 6 $\therefore \quad$ Probability of getting $1=\dfrac{1}{6}$ Now, probability of getting not $1=1-$ Probability of getting 1 $
=1-\dfrac{1}{6}=\dfrac{5}{6}
$Show Answer
Solution I toss three coins together [given] So, total number of outcomes $=2^{3}=8$ and possible outcomes are (HHH), (HTT), (THT), (TTH), (HHT), (THH), (HTH) and (TTT) Now, probability of getting no head $=\dfrac{1}{8}$ Hence, the given conclusion is wrong because the probability of no head is $\dfrac{1}{8}$ not $\dfrac{1}{4}$.Show Answer
Solution No, if let we toss a coin, then we get head or tail, both are equally likely events. So, probability is $\dfrac{1}{2}$. If we toss a coin 6 times, then probability will be same in each case. So, the probability of getting a head is not 1 .Show Answer
Solution The outcome of next toss may or may not be tail, because on tossing a coin, we get head or tail so both are equally likely events.Show Answer
Solution No, let we toss a coin, then we get head or tail, both are equaly likely events. i.e., probability of each event is $\dfrac{1}{2}$. So, no question of expecting a tail to have a higher chance in 4th toss.Show Answer
Show Answer
Solution
We know that, between 1 to 100 half numbers are even and half numbers are odd i.e., 50 numbers $(2,4,6,8, \ldots, 96,98,100)$ are even and 50 numbers $(1,3,5,7, \ldots, 97,99)$ are odd. So, both events are equally likely.
So, probability of getting even number $=\dfrac{50}{100}=\dfrac{1}{2}$
and probability of getting odd number $=\dfrac{50}{100}=\dfrac{1}{2}$
Hence, the probability of each is $\dfrac{1}{2}$.
Short Answer Type Questions
1 Find the mean of the distribution
Class | $1-3$ | $3-5$ | $5-7$ | $7-10$ |
---|---|---|---|---|
Frequency | 9 | 22 | 27 | 17 |
Solution We first, find the class mark $x_i$, of each class and then proceed as follows. Therefore, mean $(\bar{{}x})=\dfrac{\Sigma f_i x_i}{\Sigma f_i}=\dfrac{412.5}{75}=5.5$ Hence, mean of the given distribution is 5.5 .Show Answer
Class
Class marks
$(x_i)$Frequency
$(f_i)$$\boldsymbol{{}f} _{\boldsymbol{{}i}} \boldsymbol{{}x} _{\boldsymbol{{}i}}$
$1-3$
2
9
18
$3-5$
4
22
88
$5-7$
6
27
162
$7-10$
8.5
17
144.5
$\Sigma f_i=75$
$\Sigma f_i x_i=412.5$
Marks | $10-20$ | $20-30$ | $30-40$ | $40-50$ | $50-60$ |
---|---|---|---|---|---|
Number of students | 2 | 4 | 7 | 6 | 1 |
Solution We first, find the class mark $x_i$ of each class and then proceed as follows Therefore, $
\text{ mean }(\bar{{}x})=\dfrac{\Sigma f_i x_i}{\Sigma f_i}=\dfrac{700}{20}=35
$ Hence, the mean of scores of 20 students in mathematics test is 35 .Show Answer
Marks
Class marks $(x_i)$
Frequency $(f_i)$
$\boldsymbol{{}f} _{\boldsymbol{{}i}} \boldsymbol{{}x} _{\boldsymbol{{}i}}$
$10-20$
15
2
30
$20-30$
25
4
100
$30-40$
35
7
245
$40-50$
45
6
270
$50-60$
55
1
55
$\Sigma f_i=20$
$\Sigma f_i x_i=700$
Class | $4-7$ | $8-11$ | $12-15$ | $16-19$ |
---|---|---|---|---|
Frequency | 5 | 4 | 9 | 10 |
Solution Since, given data is not continuous, so we subtract 0.5 from the lower limit and add 0.5 in the upper limit of each class. Now, we first find the class mark $x_i$ of each class and then proceed as follows Therefore, $
\bar{{}x}(\text{ mean })=\dfrac{\Sigma f_i x_i}{\Sigma f_i}=\dfrac{362}{28}=12.93
$ Hence, mean of the given data is 12.93 .Show Answer
Class
Class marks $(x_i)$
Frequency $(f_i)$
$\boldsymbol{{}f} _{\boldsymbol{{}i}} \boldsymbol{{}x} _{\boldsymbol{{}i}}$
$3.5-7.5$
5.5
5
27.5
$7.5-11.5$
9.5
4
38
$11.5-15.5$
13.5
9
121.5
$15.5-19.5$
17.5
10
175
$\Sigma f_i=28$
$\boldsymbol{{}\Sigma} f_i x_i=362$
Number of pages written per day |
$16-18$ | $19-21$ | $22-24$ | $25-27$ | $28-30$ |
---|---|---|---|---|---|
Number of days | 1 | 3 | 4 | 9 | 13 |
Find the mean number of pages written per day.
Solution Since, Since, given data is not continuous, so we subtract 0.5 from the lower limit and add 0.5 in the upper limit of each class. $
\therefore \quad \text{ Mean }(\bar{{}x})=\dfrac{\Sigma f_i x_i}{\Sigma f_i}=\dfrac{780}{30}=26
$ Hence, the mean of pages written per day is 26 .Show Answer
Class mark
Mid-value $(x_i)$
Number of days $(\boldsymbol{{}f} _{\boldsymbol{{}i}})$
$\boldsymbol{{}f} _{\boldsymbol{{}i}} \boldsymbol{{}x} _{\boldsymbol{{}i}}$
$15.5-18.5$
17
1
17
$18.5-21.5$
20
3
60
$21.5-24.5$
23
4
92
$24.5-27.5$
26
9
234
27.5-30.5
29
13
377
Total
30
780
Income (in ₹) | $1-200$ | $201-400$ | $401-600$ | $601-800$ |
---|---|---|---|---|
Number of employees | 14 | 15 | 14 | 7 |
Find the mean daily income of employees.
Solution Since, given data is not continuous, so we subtract 0.5 from the lower limit and add 0.5 in the upper limit of each class. Now we first, find the class mark $x_i$ of each class and then proceed as follows $
\begin{matrix}
\therefore \quad \text{ Assumed mean, } a=300.5 \\
\text{ Class width, } h=200
\end{matrix}
$ and total observations, $N=50$ By step deviation method, $
\begin{aligned}
\text{ Mean } = a+h \times \dfrac{1}{N} \times \sum _{i=1}^{5} f_i u_i \\
& =300.5+200 \times \dfrac{1}{50} \times 14 \\
& =300.5+56=356.5
\end{aligned}
$Show Answer
Income
(in ₹)Class marks
$(x_i)$Number of
employees
$(f_i)$$\boldsymbol{{}u} _{\boldsymbol{{}i}}=\dfrac{\boldsymbol{{}x} _{\boldsymbol{{}i}}-\boldsymbol{{}a}}{\boldsymbol{{}h}}=\dfrac{\boldsymbol{{}x} _{\boldsymbol{{}i}}-\mathbf{3 0 0 . 5}}{\mathbf{2 0 0}}$
$\boldsymbol{{}f} _{\boldsymbol{{}i}} \boldsymbol{{}u} _{\boldsymbol{{}i}}$
$0.5-200.5$
100.5
14
-1
-14
$200.5-400.5$
$a=300.5$
15
0
0
$400.5-600.5$
500.5
14
1
14
$600.5-800.5$
700.5
7
2
14
$N=\Sigma f_i=50$
$\Sigma f_i u_i=14$
Number of seats | $100-104$ | $104-108$ | $108-112$ | $112-116$ | $116-120$ |
---|---|---|---|---|---|
Frequency | 15 | 20 | 32 | 18 | 15 |
Determine the mean number of seats occupied over the flights.
Solution We first, find the class mark $x_i$ of each class and then proceed as follows. $
\begin{aligned}
& \therefore \quad \text{ Assumed mean, } a=110, \\
& \text{ Class width, } h=4 \\
& \text{ and } \quad \text{ total observation, } N=100 \\
& \text{ By assumed mean method, } \\
& \text{ Mean }(\bar{{}x})=a+\dfrac{\sum f_i d_i}{\Sigma f_i} \\
&=110+\dfrac{-8}{100}=110-0.08=109.92
\end{aligned}
$Show Answer
Number of
seatsClass marks
$(x _{\boldsymbol{{}i}})$Frequency $(f_i)$
Deviation
$\boldsymbol{{}d} _{\boldsymbol{{}i}}=x _{\boldsymbol{{}i}}-\boldsymbol{{}a}$$\boldsymbol{{}f} _{\boldsymbol{{}i}} \boldsymbol{{}d} _{\boldsymbol{{}i}}$
$100-104$
102
15
-8
-120
$104-108$
106
20
-4
-80
$108-112$
$a=110$
32
0
0
$112-116$
114
18
4
72
$116-120$
118
15
8
120
$N=\Sigma f_i=100$
$\Sigma f_i d_i=-8$
Weight (in kg) | $100-110$ | $110-120$ | $120-130$ | $130-140$ | $140-150$ |
---|---|---|---|---|---|
Number of wrestlers |
4 | 14 | 21 | 8 | 3 |
Find the mean weight of the wrestlers.
Solution We first find the class mark $x_i$, of each class and then proceed as follows $\therefore$ Assumed mean $(a)=125$, Class width $(h)=10$ and total observation $(N)=50$ By assumed mean method, $
\begin{aligned}
\text{ Mean }(\bar{{}x}) = a+\dfrac{\sum f_i d_i}{\sum f_i} \\
& =125+\dfrac{(-80)}{50} \\
& =125-1.6=123.4 kg
\end{aligned}
$Show Answer
Weight
(in kg)Number of
wrestlers $(f_i)$Class marks
$(x_i)$Deviations
$\boldsymbol{{}d} _{\boldsymbol{{}i}}=x _{\boldsymbol{{}i}}-\boldsymbol{{}a}$$\boldsymbol{{}f} _{\boldsymbol{{}i}} \boldsymbol{{}d} _{\boldsymbol{{}i}}$
$100-110$
4
105
-20
-80
$110-120$
14
115
-10
-140
$120-130$
21
$a=125$
0
0
$130-140$
8
135
10
80
$140-150$
3
145
20
60
$N=\Sigma f_i=50$
$\Sigma f_i d_i=-80$
Mileage $(kmL^{-1})$ | $10-12$ | $12-14$ | $14-16$ | $16-18$ |
---|---|---|---|---|
Number of cars | 7 | 12 | 18 | 13 |
Find the mean mileage.
The manufacturer claimed that the mileage of the model was $16 kmL^{-1}$. Do you agree with this claim?
Solution Here, and $\therefore$ $
\begin{aligned}
\Sigma f_i = 50 \\
\Sigma f_i x_i = 724 \\
\text{ Mean } \bar{{}x} = \dfrac{\sum f_i x_i}{\Sigma f_i} \\
& =\dfrac{724}{50}=14.48
\end{aligned}
$ Hence, mean mileage is $14.48 kmL^{-1}$. $No$, the manufacturer is claiming mileage $1.52 kmh^{-1}$ more than average mileage.Show Answer
Mileage $(kmL^{-1})$
Class marks $(x_i)$
Number of cars
$(f_i)$$\boldsymbol{{}f} _{\boldsymbol{{}i}} \boldsymbol{{}x} _{\boldsymbol{{}i}}$
$10-12$
11
7
77
$12-14$
13
12
156
$14-16$
15
18
270
$16-18$
17
13
221
Total
$\Sigma f_i=50$
$\Sigma f_i x_i=724$
Weight (in kg) |
$40-45$ | $45-50$ | $50-55$ | $55-60$ | $60-65$ | $65-70$ | $70-75$ | $75-80$ |
---|---|---|---|---|---|---|---|---|
Number of persons |
4 | 4 | 13 | 5 | 6 | 5 | 2 | 1 |
Construct a cumulative frequency distribution (of the less than type) table for the data above.
Solution The cumulative distribution (less than type) table is shown belowShow Answer
Weight $($ in kg)
Cumulative frequency
Less than 45
4
Less than 50
$4+4=8$
Less than 55
$8+13=21$
Less than 60
$21+5=26$
Less than 65
$26+6=32$
Less than 70
$32+5=37$
Less than 75
$37+2=39$
Less than 80
$39+1=40$
Marks | Number of students |
---|---|
Below 10 | 10 |
Below 20 | 50 |
Below 30 | 130 |
Below 40 | 270 |
Below 50 | 440 |
Below 60 | 570 |
Below 70 | 670 |
Below 80 | 740 |
Below 90 | 780 |
Below 100 | 800 |
Construct a frequency distribution table for the data above.
Solution Here, we observe that 10 students have scored marks below 10 i.e., it lies between class interval 0-10. Similarly, 50 students have scored marks below 20. So, $50-10=40$ students lies in the interval $10-20$ and so on. The table of a frequency distribution for the given data isShow Answer
Class interval
Number of students
$0-10$
10
$10-20$
$50-10=40$
$20-30$
$130-50=80$
$30-40$
$270-130=140$
$40-50$
$440-270=170$
$50-60$
$570-440=130$
$60-70$
$670-570=100$
$70-80$
$740-670=70$
$80-90$
$780-740=40$
$90-100$
$800-780=20$
Marks (Out of 90) | Number of candidates |
---|---|
More than or equal to 80 | 4 |
More than or equal to 70 | 6 |
More than or equal to 60 | 11 |
More than or equal to 50 | 17 |
More than or equal to 40 | 23 |
More than or equal to 30 | 27 |
More than or equal to 20 | 30 |
More than or equal to 10 | 32 |
More than or equal to 0 | 34 |
Solution Here, we observe that, all 34 students have scored marks more than or equal to 0 . Since, 32 students have scored marks more than or equal to 10 . So, $34-32=2$ students lies in the interval 0-10 and so on. Now, we construct the frequency distribution table.Show Answer
Class interval
Number of candidates
$0-10$
$34-32=2$
$10-20$
$32-30=2$
$20-30$
$30-27=3$
$30-40$
$27-23=4$
$40-50$
$23-17=6$
$50-60$
$17-11=6$
$60-70$
$11-6=5$
$70-80$
$6-4=2$
$80-90$
4
Height $($ in cm) | Frequency | Cumulative frequency |
---|---|---|
$150-155$ | 12 | $a$ |
$155-160$ | $b$ | 25 |
$160-165$ | 10 | $c$ |
$165-170$ | $d$ | 43 |
$170-175$ | $e$ | 48 |
$175-180$ | 2 | $f$ |
Total | 50 |
Solution On comparing last two tables, we get $
\begin{aligned}
& a=12 \\
& \therefore \quad 12+b=25 \\
& \Rightarrow \quad b=25-12=13 \\
& 22+b=c \\
& c=22+13=35 \\
& 22+b+d=43 \\
& \Rightarrow \quad 22+13+d=43 \\
& \Rightarrow \quad d=43-35=8 \\
& \text{ and } 22+b+d+e=48 \\
& \Rightarrow \quad 22+13+8+e=48 \\
& \Rightarrow \quad e=48-43=5 \\
& \text{ and } \quad 24+b+d+e=f \\
& \Rightarrow \quad 24+13+8+5=f \\
& \therefore \quad f=50
\end{aligned}
$Show Answer
Height (in cm)
Frequency
Cumulative
frequency (given)Cumulative
frequency
$150-155$
12
$a$
12
$155-160$
$b$
25
$12+b$
$160-165$
10
$c$
$22+b$
$165-170$
$d$
43
$22+b+d$
$170-175$
$e$
48
$22+b+d+e$
$175-180$
2
$f$
$24+b+d+e$
Total
50
Age (in year) | $10-20$ | $20-30$ | $30-40$ | $40-50$ | $50-60$ | $60-70$ |
---|---|---|---|---|---|---|
Number of patients | 60 | 42 | 55 | 70 | 53 | 20 |
Form
(i) less than type cumulative frequency distribution.
(ii) More than type cumulative frequency distribution.
Solution (i) We observe that the number of patients which take medical treatment in a hospital on a particular day less than 10 is 0 . Similarly, less than 20 include the number of patients which take medical treatment from 0-10 as well as the number of patients which take medical treatment from 10-20. So, the total number of patients less than 20 is $0+60=60$, we say that the cumulative frequency of the class $10-20$ is 60 . Similarly, for other class. (ii) Also, we observe that all 300 patients which take medical treatment more than or equal to 10. Since, there are 60 patients which take medical treatment in the interval 10-20, this means that there are $300-60=240$ patients which take medical treatment more than or equal to 20. Continuing in the same manner.Show Answer
(i) Less than type
(ii) More than type
Age (in year)
Number of
studentsAge (in year)
Number of
students
Less than 10
0
More than or
equal to 10300
Less than 20
60
More than or
equal to 20240
Less than 30
102
More than or
equal to 30198
Less than 40
157
More than or
equal to 40143
Less than 50
227
More than or
equal to 5073
Less than 60
280
More than or
equal to 6060
Less than 70
300
Marks | Below 20 | Below 40 | Below 60 | Below 80 | Below 100 |
---|---|---|---|---|---|
Number of students |
17 | 22 | 29 | 37 | 50 |
Form the frequency distribution table for the data.
Solution Here, we observe that, 17 students have scored marks below 20 i.e., it lies between class interval 0-20 and 22 students have scored marks below 40 , so $22-17=5$ students lies in the class interval 20-40 continuting in the same manner, we get the complete frequency distribution table for given data.Show Answer
Marks
Number of students
$0-20$
17
$20-40$
$22-17=5$
$40-60$
$29-22=7$
$60-80$
$37-29=8$
$80-100$
$50-37=13$
Weekly income (in ₹) | Number of families |
---|---|
$0-1000$ | 250 |
$1000-2000$ | 190 |
$2000-3000$ | 100 |
$3000-4000$ | 40 |
$4000-5000$ | 15 |
$5000-6000$ | 5 |
Total | $\mathbf{6 0 0}$ |
Compute the median income.
Solution First we construct a cumulative frequency table. It is given that, $n=600$ $
\therefore \quad \dfrac{n}{2}=\dfrac{600}{2}=300
$ Since, cumulative frequency 440 lies in the interval $1000-2000$. Here, (lower median class) $l=1000$, $
f=190, c f=250, \text{ (class width) } h=1000
$ and (total observation) $n=600$ $
\begin{aligned}
\therefore \quad \text{ Median } = l+\dfrac{\dfrac{n}{2}-c f}{f} \times h \\
& =1000+\dfrac{(300-250)}{190} \times 1000 \\
& =1000+\dfrac{50}{190} \times 1000 \\
& =1000+\dfrac{5000}{19} \\
& =1000+263.15=1263.15
\end{aligned}
$ Hence, the median income is ₹ 1263.15 .Show Answer
Weekly income
(in ₹)Number of families
$(f_i)$Cumulative frequency
$(c f)$
$0-1000$
250
250
$1000-2000=$ mid class
$190=f$
$250+190=440$
$2000-3000$
100
$440+100=540$
$3000-4000$
40
$540+40=580$
$4000-5000$
15
$580+15=595$
$5000-6000$
5
$595+5=600$
Speed $($ in $km / h)$ | $85-100$ | $100-115$ | $115-130$ | $130-145$ |
---|---|---|---|---|
Number of players | 11 | 9 | 8 | 5 |
Calculate the median bowling speed.
Solution First we construct the cumulative frequency table It is given that, $n=33$ $
\therefore \quad \dfrac{n}{2}=\dfrac{33}{2}=16.5
$ So, the median class is $100-115$. where, $
\begin{aligned}
& \text{ lower limit }(l)=100, \\
& \text{ frequency }(f)=9
\end{aligned}
$ $
\text{ cumulative frequency }(c f)=11
$ and class width $(h)=15$ $
\begin{aligned}
\text{ Median } = l+\dfrac{\dfrac{n}{2}-c f}{f} \times h \\
& =100+\dfrac{(16.5-11)}{9} \times 15 \\
& =100+\dfrac{5.5 \times 15}{9}=100+\dfrac{82.5}{9}=100+9.17 \\
& =109.17
\end{aligned}
$ Hence, the median bowling speed is $109.17 km / h$.Show Answer
Speed (in
$km / h)$Number of
playersCumulative
frequency
$85-100$
11
11
$100-115$
9
$11+9=20$
$115-130$
8
$20+8=28$
$130-145$
5
$28+5=33$
Income (in ₹) | Number of families |
---|---|
$0-5000$ | 8 |
$5000-10000$ | 26 |
$10000-15000$ | 41 |
$15000-20000$ | 16 |
$20000-25000$ | 3 |
$25000-30000$ | 3 |
$30000-35000$ | 2 |
$35000-40000$ | 1 |
Calculate the modal income.
Solution In a given data, the highest frequency is 41, which lies in the interval 10000-15000. Here, $l=10000, f_m=41, f_1=26, f_2=16$ and $h=5000$ $
\begin{aligned}
\therefore \quad \text{ Mode } = l+\dfrac{f_m-f_1}{2 f_m-f_1-f_2} \times h \\
& =10000+\dfrac{41-26}{2 \times 41-26-16} \times 5000 \\
& =10000+\dfrac{15}{82-42} \times 5000 \\
& =10000+\dfrac{15}{40} \times 5000 \\
& =10000+15 \times 125=10000+1875=\text{₹} 11875
\end{aligned}
$ Hence, the modal income is ₹ 11875Show Answer
Weight (in g) | Number of packets |
---|---|
$200-201$ | 12 |
$201-202$ | 26 |
$202-203$ | 20 |
$203-204$ | 9 |
$204-205$ | 2 |
$205-206$ | 1 |
Determine the model weight.
Solution In the given data, the highest frequency is 26 , which lies in the interval 201-202 Here, $
l=201, f_m=26, f_1=12, f_2=20 \text{ and (class width) } h=1
$ $
\begin{aligned}
\therefore \quad \text{ Mode } = l+\dfrac{f_m-f_1}{2 f_m-f_1-f_2} \quad \times h=201+\dfrac{26-12}{2 \times 26-12-20} \times 1 \\
=201+\dfrac{14}{52-32}=201+\dfrac{14}{20}=201+0.7=201.7 g
\end{aligned}
$ Hence, the modal weight is $201.7 g$.Show Answer
(i) same number on both dice.
(ii) different number on both dice.
Solution Two dice are thrown at the same time. [given] So, total number of possible outcomes $=36$ (i) We have, same number on both dice. So, possible outcomes are $(1,1),(2,2),(3,3),(4,4),(5,5)$ and $(6,6)$. $\therefore$ Number of possible outcomes $=6$ Now, required probability $=\dfrac{6}{36}=\dfrac{1}{6}$
(ii) We have, different number on both dice. So, number of possible outcomes $=36-$ Number of possible outcomes for same number on both dice $
=36-6=30
$ $\therefore$ Required probability $=\dfrac{30}{36}=\dfrac{5}{6}$Show Answer
Solution Two dice are thrown simultaneously. [given] So, total number of possible outcomes $=36$ (i) Sum of the numbers appearing on the dice is 7 . So, the possible ways are $(1,6),(2,5),(3,4),(4,3),(5,2)$ and $(6,1)$. Number of possible ways $=6$ $\therefore$ Required probability $=\dfrac{6}{36}=\dfrac{1}{6}$ (ii) Sum of the numbers appearing on the dice is a prime number i.e., 2, 3, 5, 7 and 11. So, the possible ways are $(1,1),(1,2),(2,1),(1,4),(2,3),(3,2),(4,1),(1,6),(2,5)$, $(3,4),(4,3),(5,2),(6,1),(5,6)$ and $(6,5)$. Number of possible ways $=15$ $\therefore$ Required probability $=\dfrac{15}{36}=\dfrac{5}{12}$ (iii) Sum of the numbers appearing on the dice is 1. It is not possible, so its probability is zero.Show Answer
Solution Number of total outcomes $=36$ (i) When product of the numbers on the top of the dice is 6 . So, the possible ways are $(1,6),(2,3),(3,2)$ and $(6,1)$. Number of possible ways $=4$ $\therefore \quad$ Required probability $=\dfrac{4}{36}=\dfrac{1}{9}$ (ii) When product of the numbers on the top of the dice is 12 . So, the possible ways are $(2,6),(3,4),(4,3)$ and $(6,2)$. Number of possible ways $=4$ $\therefore \quad$ Required probability $=\dfrac{4}{36}=\dfrac{1}{9}$ (iii) Product of the numbers on the top of the dice cannot be 7 . So, its probability is zero.Show Answer
Solution Number of total outcomes $=36$ When product of numbers appearing on them is less than 9 , then possible ways are $(1,6)$, $(1,5)(1,4),(1,3),(1,2),(1,1),(2,2),(2,3),(2,4),(3,2),(4,2),(4,1),(3,1),(5,1),(6,1)$ and $(2,1)$. Number of possible ways $=16$ $\therefore$ Required probability $=\dfrac{16}{36}=\dfrac{4}{9}$Show Answer
Solution Number of total outcomes $=n(s)=36$ (i) Let $E_1=$ Event of getting sum $2={(1,1),(1,1)}$ $
\begin{matrix} & n(E_1)=2 \\
\therefore & P(E_1)=\dfrac{n(E_1)}{n(S)}=\dfrac{2}{36}=\dfrac{1}{18}
\end{matrix}
$ (ii) Let $E_2=$ Event of getting sum $3={(1,2),(1,2),(2,1),(2,1)}$ $
\begin{aligned}
& n(E_2)=4 \\
& \therefore \quad P(E_2)=\dfrac{n(E_2)}{n(S)}=\dfrac{4}{36}=\dfrac{1}{9}
\end{aligned}
$ (iii) Let $E_3=$ Event of getting sum $4={(2,2),(2,2),(3,1),(3,1),(1,3),(1,3)}$ $
\begin{matrix}
\therefore & n(E_3)=6 \\
\therefore & P(E_3)=\dfrac{n(E_3)}{n(S)}=\dfrac{6}{36}=\dfrac{1}{6}
\end{matrix}
$ (iv) Let $E_4=$ Event of getting sum $5={(2,3),(2,3),(4,1),(4,1),(3,2),(3,2)}$ $
\begin{matrix}
\therefore & n(E_4)=6 \\
\therefore & P(E_4)=\dfrac{n(E_4)}{n(S)}=\dfrac{6}{36}=\dfrac{1}{6}
\end{matrix}
$ (v) Let $E_5=$ Event of getting sum $6={(3,3),(3,3),(4,2),(4,2),(5,1),(5,1)}$ $
\begin{matrix}
n(E_5) = 6 \\
\therefore & P(E_5)=\dfrac{n(E_5)}{n(S)}=\dfrac{6}{36}=\dfrac{1}{6}
\end{matrix}
$ (vi) Let $E_6=$ Event of getting sum $7={(4,3),(4,3),(5,2),(5,2),(6,1),(6,1)}$ $
\begin{matrix}
\therefore & n(E_6)=6 \\
\therefore & P(E_6)=\dfrac{n(E_6)}{n(S)}=\dfrac{6}{36}=\dfrac{1}{6}
\end{matrix}
$ (vii) Let $E_7=$ Event of getting sum $8={(5,3),(5,3),(6,2),(6,2)}$ $
\begin{matrix}
\therefore & n(E_7)=4 \\
\therefore & P(E_7)=\dfrac{n(E_7)}{n(S)}=\dfrac{4}{36}=\dfrac{1}{9}
\end{matrix}
$ (viii) Let $E_8=$ Event of getting sum $9={(6,3),(6,3)}$ $
\begin{matrix}
\therefore & n(E_8)=2 \\
\therefore & P(E_8)=\dfrac{n(E_8)}{n(S)}=\dfrac{2}{36}=\dfrac{1}{18}
\end{matrix}
$Show Answer
Solution The possible outcomes, if a coin is tossed 2 times is $
\begin{aligned}
& \therefore = {(H H),(T T),(H T),(T H)} \\
& n(S) = 4
\end{aligned}
$ Let $E=$ Event of getting atmost one head $
\begin{matrix} ={(T T),(H T),(T H)} \\
\therefore \quad n(E) = 3 \\
\text{ Hence, } \quad \text{ required probability } = \dfrac{n(E)}{n(S)}=\dfrac{3}{4}
\end{matrix}
$Show Answer
Solution The possible outcomes if a coin is tossed 3 times is $
\begin{aligned}
& S={(H H H),(T T T),(H T T),(T H T),(T T H),(T H H),(H T H),(H H T)} \\
& \therefore \quad n(S)=8
\end{aligned}
$ (i) Let $E_1=$ Event of getting all heads $={(H H H)}$ $
\begin{matrix}
\therefore & n(E_1)=1 \\
\therefore & P(E_1)=\dfrac{n(E_1)}{n(S)}=\dfrac{1}{8}
\end{matrix}
$ (ii) Let $E_2=$ Event of getting atleast 2 heads $={(H H T),(H T H),(T H H),(H H H)}$ $
\begin{matrix}
\therefore & n(E_2)=4 \\
\therefore & P(E_2)=\dfrac{n(E_2)}{n(S)}=\dfrac{4}{8}=\dfrac{1}{2}
\end{matrix}
$Show Answer
Solution The total number of sample space in two dice, $n(S)=6 \times 6=36$ $
\begin{aligned}
& \text{ Let } \quad E=\text{ Event of getting the numbers whose difference is } 2 \\
& ={(1,3),(2,4),(3,5),(4,6),(3,1),(4,2),(5,3),(6,4)} \\
& \therefore \quad n(E)=8 \\
& \therefore \quad P(E)=\dfrac{n(E)}{n(S)}=\dfrac{8}{36}=\dfrac{2}{9}
\end{aligned}
$Show Answer
Solution If a ball is drawn out of 22 balls ( 5 blue +7 green +10 red), then the total number of outcomes are $
n(S)=22
$ (i) Let $E_1=$ Event of getting a red ball $
\begin{matrix}
n(E_1)=10 \\
\therefore \quad \text{ Required probability }=\dfrac{n(E_1)}{n(S)}=\dfrac{10}{22}=\dfrac{5}{11}
\end{matrix}
$ (ii) Let $E_2=$ Event of getting a green ball $
\begin{aligned}
n(E_2) = 7 \\
\therefore \quad \text{ Required probability }= & \dfrac{n(E_2)}{n(S)}=\dfrac{7}{22}
\end{aligned}
$ (iii) Let $E_3=$ Event getting a red ball or a green ball i.e., not a blue ball. $
\begin{aligned}
& n(E_3)=(10+7)=17 \\
& \therefore \quad \text{ Required probability }=\dfrac{n(E_3)}{n(S)}=\dfrac{17}{22}
\end{aligned}
$Show Answer
Solution If we remove one king, one queen and one jack of clubs from 52 cards, then the remaining cards left, $n(S)=49$ (i) Let $E_1=$ Event of getting a heart $
\begin{aligned}
& n(E_1)=13 \\
\therefore & P(E_1)=\dfrac{n(E_1)}{n(S)}=\dfrac{13}{49}
\end{aligned}
$ (ii) Let $E_2=$ Event of getting a king $n(E_2)=3$ [since, out of 4 king, one club cards is already removed] $\therefore \quad P(E_2)=\dfrac{n(E_2)}{n(S)}=\dfrac{3}{49}$Show Answer
Solution (i) $L$ et $E_3=$ Event of getting a club $n(E_3)=(13-3)=10$ $\therefore$ Required probability $=\dfrac{n(E_3)}{n(S)}=\dfrac{10}{49}$ (ii) Let $E_4=$ Event of getting 10 of hearts $
n(E_4)=1
$ [because in 52 playing cards only 13 are the heart cards and only one 10 in 13 heart cards] $\therefore \quad$ Required probability $=\dfrac{n(E_4)}{n(S)}=\dfrac{1}{49}$Show Answer
Solution In out of 52 playing cards, 4 jacks, 4 queens and 4 kings are removed, then the remaining cards are left, $n(S)=52-3 \times 4=40$. (i) Let $E_1=$ Event of getting a card whose value is 7 $E=$ Card value 7 may be of a spade, a diamond, a club or a heart $
\begin{matrix}
\therefore & n(E_1)=4 \\
\therefore & P(E_1)=\dfrac{n(E_1)}{n(S)}=\dfrac{4}{40}=\dfrac{1}{10}
\end{matrix}
$ (ii) Let $E_2=$ Event of getting a card whose value is greater than 7 $=$ Event of getting a card whose value is 8,9 or 10 $
\begin{matrix}
\therefore & n(E_2)=3 \times 4=12 \\
\therefore & P(E_2)=\dfrac{n(E_2)}{n(S)}=\dfrac{12}{40}=\dfrac{3}{10}
\end{matrix}
$ (iii) Let $E_3=$ Event of getting a card whose value is less than 7 $=$ Event of getting a card whose value is 1, 2, 3, 4, 5 or 6 $
\begin{matrix}
\therefore & n(E_3)=6 \times 4=24 \\
\therefore & P(E_3)=\dfrac{n(E_3)}{n(S)}=\dfrac{24}{40}=\dfrac{3}{5}
\end{matrix}
$Show Answer
Solution The number of integers between 0 and 100 is $
n(S)=99
$ (i) Let $E_1=$ Event of choosing an integer which is divisible by 7 $=$ Event of choosing an integer which is multiple of 7 $={7,14,21,28,35,42,49,56,63,70,77,84,91,98}$ $
\begin{matrix}
\therefore & n(E_1)=14 \\
\therefore & P(E_1)=\dfrac{n(E_1)}{n(S)}=\dfrac{14}{99}
\end{matrix}
$ (ii) Let $E_2=$ Event of choosing an integer which is not divisible by 7 $
\begin{matrix}
\therefore & n(E_2) = n(S)-n(E_1) \\
& =99-14=85 \\
\therefore & P(E_2) = \dfrac{n(E_2)}{n(S)}=\dfrac{85}{99}
\end{matrix}
$Show Answer
Solution Total number of out comes with numbers 2 to $101, n(s)=100$ (i) Let $E_1=$ Event of selecting a card which is an even number $={2,4,6, \ldots 100}$ $
\begin{array}{ll} [\text{ in an AP, } l=a+(n-1) d, \text{ here } l = 100, a=2 \text{ and } d=2 \Rightarrow 100=2+(n-1) 2 \\
\Rightarrow (n-1)=49 \Rightarrow n=50] \\
\therefore n(E_1)=50 \\
\therefore \text{Requied probability} = \dfrac{n(E_1)}{n(S)}=\dfrac{50}{100}=\dfrac{1}{2}
\end{array}
$ (ii) Let $E_2=$ Event of selecting a card which is a square number $
\begin{aligned}
& \begin{matrix}
={4,9,16,25,36,49,64,81,100} \\
={(2)^{2},(3)^{2},(4)^{2},(5)^{2},(6)^{2},(7)^{2},(8)^{2},(9)^{2},(10)^{2}} \\
\end{matrix} \\
\therefore & \quad \quad n(E_2)=9 \\
& \text{ Hence, required probability }=\dfrac{n(E_2)}{n(S)}=\dfrac{9}{100}
\end{aligned}
$Show Answer
Solution We know that, in english alphabets, there are ( 5 vowels +21 consonants) $=26$ letters. So, total number of outcomes in english alphabets are, $
\begin{aligned}
& n(S)=26 \\
& \text{ Let } \quad E=\text{ Event of choosing a english alphabet, which is a consonent } \\
& ={b, c, d, f, g, h, j, k, l, m, n, p, q, r, s, t, v, w, x, y, z} \\
& \therefore \quad n(E)=21
\end{aligned}
$ Hence, required probability $=\dfrac{n(E)}{n(S)}=\dfrac{21}{26}$Show Answer
Solution Total number of sealed envelopes in a box, $n(S)=1000$ Number of envelopes containing cash prize $=10+100+200=310$ Number of envelopes containing no cash prize, $
\begin{matrix} & n(E)=1000-310=690 \\
\therefore & P(E)=\dfrac{n(E)}{n(S)}=\dfrac{690}{1000}=\dfrac{69}{100}=0.69
\end{matrix}
$Show Answer
Solution Total number of slips in a box, $n(S)=25+50=75$ From the chart it is clear that, there are 11 slips which are marked other than ₹ 1 . $\therefore \quad$ Required probability $=\dfrac{\text{ Number of slips other than ₹ } 1}{\text{ Total number of slips }}=\dfrac{11}{75}$Show Answer
Solution $\because$ Total number of bulbs, $n(S)=24$ Let $E_1=$ Event of selecting not defective bulb $=$ Event of selecting good bulbs $
\begin{aligned}
& n(E_1)=18 \\
\therefore & P(E_1)=\dfrac{n(E_1)}{n(S)}=\dfrac{18}{24}=\dfrac{3}{4}
\end{aligned}
$ Suppose, the selected bulb is defective and not replaced, then total number of bulbs remains in a carton, $n(S)=23$. In them, 18 are good bulbs and 5 are defective bulbs. $\therefore P($ selecting second defective bulb $)=\dfrac{5}{23}$Show Answer
Solution Total number of figures $
n(S)=8 \text{ triangles }+10 \text{ squares }=18
$ (i) $P$ (lost piece is a triangle) $=\dfrac{8}{18}=\dfrac{4}{9}$ (ii) $P$ (lost piece is a square) $=\dfrac{10}{18}=\dfrac{5}{9}$ (iii) $P$ (square of blue colour) $=\dfrac{6}{18}=\dfrac{1}{3}$ (iv) $P$ (triangle of red colour) $=\dfrac{5}{18}$Show Answer
(i) loses the entry fee.
(ii) gets double entry fee.
(iii) just gets her entry fee.
Solution Total possible outcomes of tossing a coin 3 times,
$S={(H H H),(T T T),(H T T),(T H T),(T T H),(T H H),(H T H),(H H T)}$
$n(S)=8$ (i) Let $E_1=$ Event that Sweta losses the entry fee $=$ She tosses tail on three times $n(E_1)={(T T T)}$ $P(E_1)=\dfrac{n(E_1)}{n(S)}=\dfrac{1}{8}$ (ii) Let $E_2=$ Event that Sweta gets double entry fee $=$ She tosses heads on three times $={(H H H)}$ $
\begin{aligned}
& n(E_2)=1 \\
\therefore & P(E_2)=\dfrac{n(E_2)}{n(S)}=\dfrac{1}{8}
\end{aligned}
$ (iii) Let $E_3=$ Event that Sweta gets her entry fee back $
\begin{aligned}
& =\text{ Sweta gets heads one or two times } \\
& ={(H T T),(T H T),(T T H),(H H T),(H T H),(T H H)} \\
& \therefore \quad n(E_3)=6 \\
& \therefore \quad P(E_3)=\dfrac{n(E_3)}{n(S)}=\dfrac{6}{8}=\dfrac{3}{4}
\end{aligned}
$Show Answer
(i) How many different scores are possible?
(ii) What is the probability of getting a total of 7 ?
Solution Given, a die has its six faces marked ${0,1,1,1,6,6}$ $\therefore$ Total sample space, $n(S)=6^{2}=36$ (i) The different score which are possible are 6 scores i.e., 0,1,2,6,7 and12. (ii) Let $E=$ Event of getting a sum 7 $
\begin{aligned}& =\{(1,6),(1,6),(1,6),(1,6),(1,6),(1,6),(6,1),(6,1),(6,1),(6,1),(6,1),(6,1)\} \\
& \therefore \quad n(E)=12 \\
& \therefore \quad P(E)=\dfrac{n(E)}{n(S)}=\dfrac{12}{36}=\dfrac{1}{3}
\end{aligned}
$Show Answer
(i) acceptable to Varnika?
(ii) acceptable to the trader?
Solution Given, total number of mobile phones $
n(S)=48
$ (i) Let $E_1=$ Event that Varnika will buy a mobile phone $=$ Varnika buy only, if it is good mobile $\therefore n(E_1)=42$ $\therefore P(E_1)=\dfrac{n(E_1)}{n(S)}=\dfrac{42}{48}=\dfrac{7}{8}$ (ii) Let $E_2=$ Event that trader will buy only when it has no major defects $=$ Trader will buy only 45 mobiles $
\begin{matrix}
\therefore & n(E_2)=45 \\
\therefore & P(E_2)=\dfrac{n(E_2)}{n(S)}=\dfrac{45}{48}=\dfrac{15}{16}
\end{matrix}
$Show Answer
Solution Given that, A bag contains total number of balls $=24$ A bag contains number of red balls $=x$ A bag contains number of white balls $=2 x$ and a bag contains number of blue balls $=3 x$ By condition, $\quad x+2 x+3 x=24$ $\Rightarrow \quad 6 x=24$ $\therefore \quad x=4$
$\therefore$ Number of red balls $=x=4$ Number of white balls $=2 x=2 \times 4=8$ and number of blue balls $=3 x=3 \times 4=12$ So, total number of outcomes for a ball is selected at random in a bag contains 24 balls. $\Rightarrow \quad n(S)=24$ (i) Let $E_1=$ Event of selecting a ball which is not red i.e., can be white or blue. $\therefore n(E_1)=$ Number of white balls + Number of blue balls $ \Rightarrow \hspace{10 mm} n(E_1) =8+12=20 \\
\therefore \hspace{10 mm} \text{Requied probability} = \dfrac{n(E_1)}{n(S)}=\dfrac{20}{24}=\dfrac{5}{6}
$ (ii) Let $E_2=$ Event of selecting a ball which is white $\therefore n(E_2)=$ Number of white balls $=8$ So, required probability $=\dfrac{n(E_2)}{n(S)}=\dfrac{8}{24}=\dfrac{1}{3}$Show Answer
(i) the first player wins a prize?
(ii) the second player wins a prize, if the first has won?
Show Answer
Solution
Given that, at a fete, cards bearing numbers 1 to 1000 one number on one card, are put in a box. Each player selects one card at random and that card is not replaced so, the total number of outcomes are $n(S)=1000$
If the selected card has a perfect square greater than 500 , then player wins a prize.
(i) Let $E_1=$ Event first player wins a prize $=$ Player select a card which is a perfect square greater than 500
$={529,576,625,676,729,784,841,900,961}$
$={(23)^{2},(24)^{2},(25)^{2},(26)^{2},(27)^{2},(28)^{2},(29)^{2},(30)^{2},(31)^{2}}$
$\therefore \quad n(E_1)=9$
So, required probability $=\dfrac{n(E_1)}{n(S)}=\dfrac{9}{1000}=0.009$
(ii) First, has won i.e., one card is already selected, greater than 500 , has a perfect square. Since, repeatition is not allowed. So, one card is removed out of 1000 cards. So, number of remaining cards is 999 .
$\therefore$ Total number of remaining outcomes, $n(S^{\prime})=999$
Let $E_2=$ Event the second player wins a prize, if the first has won.
$=$ Remaining cards has a perfect square greater than 500 are 8 .
$\therefore \quad n(E_2)=9-1=8$
So, required probability $=\dfrac{n(E_2)}{n(S^{\prime})}=\dfrac{8}{999}$
Long Answer Type Questions
1 Find the mean marks of students for the following distribution
Marks | Number of students |
---|---|
0 and above | 80 |
10 and above | 77 |
20 and above | 72 |
30 and above | 65 |
40 and above | 55 |
50 and above | 43 |
60 and above | 28 |
70 and above | 16 |
80 and above | 10 |
90 and above | 8 |
100 and above | 0 |
SolutionShow Answer
Marks
Class marks
$(x_i)$Number of students
(Cumulative frequency)$\boldsymbol{{}f} _{\boldsymbol{{}i}}$
$\boldsymbol{{}f} _{\boldsymbol{{}i}} \boldsymbol{{}x} _{\boldsymbol{{}i}}$
$0-10$
5
80
3
15
$10-20$
15
77
5
75
$20-30$
25
72
7
175
$30-40$
35
65
10
350
$40-50$
45
55
12
540
$50-60$
55
43
15
825
$60-70$
65
28
12
780
$70-80$
75
16
6
450
$80-90$
85
10
2
170
$90-100$
95
8
8
760
$100-110$
105
0
0
0
Marks | Number of students |
---|---|
Below 10 | 5 |
Below 20 | 9 |
Below 30 | 17 |
Below 40 | 29 |
Below 50 | 45 |
Below 60 | 60 |
Below 70 | 70 |
Below 80 | 78 |
Below 90 | 83 |
Below 100 | 85 |
Solution Here, we observe that, 5 students have scored marks below 10, i.e. it lies between class interval 0-10 and 9 students have scored marks below 20. So, $(9-5)=4$ students lies in the class interval $10-20$. Continuing in the same manner, we get the complete frequency distribution table for given data. Here, (assumed mean) $a=45$ and (class width) $h=10$ By step deviation method, $
\begin{aligned}
\text{ Mean }(\bar{{}x}) = a+\dfrac{\sum f_i u_i}{\sum f_i} \times h=45+\dfrac{29}{85} \times 10=45+\dfrac{58}{17} \\
& =45+3.41=48.41
\end{aligned}
$Show Answer
Marks
Number of
students $(f_i)$Class marks
$(x_i)$$\boldsymbol{{}u} _{\boldsymbol{{}i}}=\dfrac{x _{\boldsymbol{{}i}}-\boldsymbol{{}a}}{\boldsymbol{{}h}}=\dfrac{x _{\boldsymbol{{}i}}-\mathbf{4 5}}{\boldsymbol{{}h}}$
$\boldsymbol{{}f} _{\boldsymbol{{}i}} \boldsymbol{{}u} _{\boldsymbol{{}i}}$
$0-10$
5
5
-4
-20
$10-20$
$9-5=4$
15
-3
-12
$20-30$
$17-9=8$
25
-2
-16
$30-40$
$29-17=12$
35
-1
-12
$40-50$
$45-29=16$
$a=45$
0
0
$50-60$
$60-45=45$
55
1
15
$60-70$
$70-60=10$
65
2
20
$70-80$
$78-70=8$
75
3
24
$80-90$
$83-78=5$
85
4
20
$90-100$
$85-83=2$
95
5
10
$N=\sum f_i=85$
$\bar{{}\sum} f_i u_i=29$
Age equal and above (in years) |
0 | 10 | 20 | 30 | 40 | 50 | 60 | 70 |
---|---|---|---|---|---|---|---|---|
Number of persons |
100 | 90 | 75 | 50 | 25 | 15 | 5 | 0 |
Solution Here, we observe that, all 100 residents of a town have age equal and above 0. Since, 90 residents of a town have age equal and above 10. So, $100-90=10$ residents lies in the interval 0-10 and so on. Continue in this manner, we get frequency of all class intervals. Now, we construct the frequency distribution table. Here, (assumed mean) $a=35$ and (class width) $h=10$ By step deviation method, $
\begin{aligned}
Mean(\bar{{}x}) = a+\dfrac{\sum f_i u_i}{\sum f_i} \times h \\
& =35+\dfrac{(-40)}{100} \times 10 \\
& =35-4=31 .
\end{aligned}
$ Hence, the required mean age is $31 yr$.Show Answer
Class interval
Number of
persons $(f_i)$Class marks
$(x_i)$$\boldsymbol{{}u} _{\boldsymbol{{}i}}=\dfrac{x _{\boldsymbol{{}i}}-\boldsymbol{{}a}}{\boldsymbol{{}h}}$
$\boldsymbol{{}f} _{\boldsymbol{{}i}} \boldsymbol{{}u} _{\boldsymbol{{}i}}$
$0-10$
$100-90=10$
5
-3
-30
$10-20$
$90-75=15$
15
-2
-30
$20-30$
$75-50=25$
25
-1
-25
$30-40$
$50-25=25$
$35=a$
0
0
$40-50$
$25-15=10$
45
1
10
$50-60$
$15-5=10$
55
2
20
$60-70$
$5-0=5$
65
3
15
$N=\sum f_i=100$
$\sum f_i u_i=-40$
Weight (in g) | Number of packets |
---|---|
$200-201$ | 13 |
$201-202$ | 27 |
$202-203$ | 18 |
$203-204$ | 10 |
$204-205$ | 1 |
$205-206$ | 1 |
Find the mean weight of packets.
Solution First, we find the class marks of the given data as follows. Here, (assume mean) a $=203.5$ and (class width) $\quad h=1$ By assumed mean method, $
\begin{aligned}
Mean(\bar{{}x}) = a+\dfrac{\sum f_i d_i}{\sum f_i} \\
& =203.5-\dfrac{108}{70} \\
& =203.5-1.54=201.96
\end{aligned}
$ Hence, the required mean weight is $201.96 g$.Show Answer
Weight
(in g)Number of
Packets $(f_i)$Class marks
$(x_i)$Deviation
$(d_i=x_i-a)$$\boldsymbol{{}f} _{\boldsymbol{{}i}} \boldsymbol{{}d} _{\boldsymbol{{}i}}$
$200-201$
13
200.5
-3
-39
$201-202$
27
201.5
-2
-54
$202-203$
18
202.5
-1
-18
$203-204$
10
$a=203.5$
0
0
$204-205$
1
204.5
1
1
$205-206$
1
205.5
2
2
$N=\sum f_i=70$
$\sum f_i d_i=-108$
Solution We observe that, the number of packets less than 200 is 0 . Similarly, less than 201 include the number of packets from 0-200 as well as the number of packets from 200-201. So, the total number of packets less than 201 is $0+13=13$. We say that, the cumulative frequency of the class $200-201$ is 13 . Similarly, for other class. To draw the less than type ogive, we plot the points $(200,0),(201,13),(202,40)(203,58)$, $(204,68),(205,69)$ and $(206,70)$ on the paper and join by free hand. $\because$ Total number of packets $(n)=70$ Now, Firstly, we plot a point $(0,35)$ on $Y$-axis and draw a line $y=35$ parallel to $X$-axis. The line cuts the less than ogive curve at a point. We draw a line on that point which is perpendicular to $X$-axis. The foot of the line perpendicular to $X$-axis is the required median. $\therefore$ Median weight $=201.8 g$Show Answer
Less than type
Weight (in g)
Number of packets
Less than 200
0
Less than 201
$0+13=13$
Less than 202
$27+13=40$
Less than 203
$18+40=58$
Less than 204
$10+58=68$
Less than 205
$1+68=69$
Less than 206
$1+69=70$
Solution For less than type table we follow the Q.5. Here, we observe that, the weight of all 70 packets is more than or equal to 200 . Since, 13 packets lie in the interval 200-201. So, the weight of $70-13=57$ packets is more than or equal to 201. Continuing in this manner we will get remaining more than or equal to 202, 203, 204, 205 and 206. To draw the less than type ogive, we plot the points $(200,0),(201,13),(202,40),(203,58)$, $(204,68),(205,69),(206,70)$ on the paper and join them by free hand. To draw the more than type ogive plot the points $(200,70),(201,57),(202,30),(203,12)$, $(204,2),(205,1),(206,0)$ on the the graph paper and join them by free hand. Hence, required median weight $=I$ ntersection point of $X$-axis $=201.8 g$.Show Answer
(i) Less than type
(ii) More than type
Weight (in g)
Number of
packetsNumber of packets
Number of
students
Less than 200
0
More than or equal to 200
70
Less than 201
13
More than or equal to 201
$70-13=57$
Less than 202
40
More than or equal to 202
$57-27=30$
Less than 203
58
More than or equal to 203
$30-18=12$
Less than 204
68
More than or equal to 204
$12-10=2$
Less than 205
69
More than or equal to 205
$2-1=1$
Less than 206
70
More than or equal to 206
$1-1=0$
Salary (in ₹ thousand) | Number of persons |
---|---|
$5-10$ | 49 |
$10-15$ | 133 |
$15-20$ | 63 |
$20-25$ | 15 |
$25-30$ | 6 |
$30-35$ | 7 |
$35-40$ | 4 |
$40-45$ | 2 |
$45-50$ | 1 |
Calculate the median and mode of the data.
Solution First, we construct a cumulative frequency table (i) Here, median class is $10-15$, because 140 lies in it. Lower limit $(l)=10$, Frequency $(f)=133$, Cumulative frequency $(c f)=49$ and class width $(h)=5$ $
\therefore \quad \begin{aligned}
\text{ Median } = l+\dfrac{\dfrac{N}{2}-c f}{f} \times h \\
& =10+\dfrac{(140-49)}{133} \times 5 \\
& =10+\dfrac{91 \times 5}{133} \\
& =10+\dfrac{455}{133}=10+3.421 \\
& =\text{₹} 13.421(\text{ in thousand) } \\
& =13.421 \times 1000 \\
& =\text{₹} 13421
\end{aligned}
$ (ii) Here, the highest frequency is 133, which lies in the interval 10-15, called modal class. Lower $limit(l)=10$, class width $(h)=5, f_m=133, f_1=49$, and $f_2=63$. $
\begin{array}{rl}
\therefore \quad \text{ Mode } & = l+\dfrac{f_m-f_1}{2 f_m-f_1-f_2} \times h \\
& =10+\dfrac{133-49}{2 \times 133-49-63} \times 5 \\
& =10+\dfrac{84 \times 5}{266-112}=10+\dfrac{84 \times 5}{154}=10+2.727 \\
& =\text{₹} 12.727 \text{ (in thousand) } \\
& =12.727 \times 1000=\text{₹} 12727
\end{array}
$ Hence, the median and modal salary are ₹ 13421 and ₹ 12727, respectively.Show Answer
Salary (in $\text{₹}$ thousand)
Number of persons $(f_i)$
Cumulative frequency $(c f)$
$5-10$
$49=f_1$
$49=c f$
$10-15$
$f_m=133=f$
$133+49=182$
$15-20$
$63=f_2$
$182+63=245$
$20-25$
15
$245+15=260$
$25-30$
6
$260+6=266$
$30-35$
7
$266+7=273$
$35-40$
4
$273+4=277$
$40-45$
2
$277+2=279$
$45-50$
1
$279+1=280$
$\therefore$
$N=280$
Class | $0-20$ | $20-40$ | $40-60$ | $60-80$ | $80-100$ |
---|---|---|---|---|---|
Frequency | 17 | $f_1$ | 32 | $f_2$ | 19 |
Solution First we calculate the class mark of given data Given that, sum of all frequencies $=120$ $
\begin{matrix}
\Rightarrow & \sum f_i=68+f_1+f_2=120 \\
\Rightarrow & f_1+f_2=52 \\
\text{ Here, } & \text{ (assumed mean) } a=50 \\
\text{ and } & \text{ (class width) } h=20
\end{matrix}
$ By step deviation method, $$
\begin{matrix}
\text{ Mean } = a+\dfrac{\sum f_i u_i}{\sum f_i} \times h \\
\Rightarrow & 50 = 50+\dfrac{(4+f_2-f _{1)}.}{120} \times 20 \\
\Rightarrow & 4+f_2-f_1 = 0 \\
\Rightarrow & -f_2+f_1 = 4 \tag{ii}
\end{matrix}
$$ On adding Eqs. (i) and (ii), we get $
\begin{matrix}
2 & 2 f_1 = 56 \\
\Rightarrow & f_1 = 28
\end{matrix}
$ Put the value of $f_1$ in Eq. (i), we get $
\begin{aligned}
& f_2=52-28 \\
& f_2=24
\end{aligned}
$ Hence, $f_1=28$ and $f_2=24$.Show Answer
Class
Frequency $(f_i)$
Class marks $(x_i)$
$\boldsymbol{{}u} _{\boldsymbol{{}i}}=\dfrac{x _{\boldsymbol{{}i}}-\boldsymbol{{}a}}{\boldsymbol{{}h}}$
$\boldsymbol{{}f} _{\boldsymbol{{}i}} \boldsymbol{{}u} _{\boldsymbol{{}i}}$
$0-20$
17
10
-2
-34
$20-40$
$f_1$
30
-1
$-f_1$
$40-60$
32
$a=50$
0
0
$60-80$
$f_2$
70
1
$f_2$
$80-100$
19
90
2
38
$\sum f_i=68+f_1+f_2$
$\sum f_i u_i=4+f_2-f_1$
Marks | Frequency |
---|---|
$20-30$ | $p$ |
$30-40$ | 15 |
$40-50$ | 25 |
$50-60$ | 20 |
$60-70$ | 9 |
$70-80$ | 8 |
$80-90$ | 10 |
Solution Given, $
N=90
$ $
\therefore \quad \dfrac{N}{2}=\dfrac{90}{2}=45
$ which lies in the interval 50-60. Lower limit, $l=50, f=20, c f=40+p, h=10$ $
\begin{matrix}
\therefore & \text{ Median } = l+\dfrac{\dfrac{N}{2}-c f}{f} \times h \\
& = 50+\dfrac{(45-40-p)}{20} \times 10 \\
\Rightarrow & 50 = 50+\dfrac{5-p}{2} \\
\Rightarrow & 0 = \dfrac{5-p}{2} \\
\therefore & p = 5 \\
\text{ Also, } & 78+p+q = 90 \\
\Rightarrow & 78+5+ & q = 90 \\
\Rightarrow & q = 90-83 \\
\therefore & q = 7
\end{matrix}
$Show Answer
Marks
Frequency
Cumulative
frequency
$20-30$
$p$
$p$
$30-40$
15
$15+p$
$40-50$
25
$40+p=c f$
$50-60$
$20=f$
$60+p$
$60-70$
$q$
$60+p+q$
$70-80$
8
$68+p+q$
$80-90$
10
$78+p+q$
Height (in cm) | Number of children |
---|---|
$124-128$ | 5 |
$128-132$ | 8 |
$132-136$ | 17 |
$136-140$ | 24 |
$140-144$ | 16 |
$144-148$ | 12 |
$148-152$ | 6 |
$152-156$ | 4 |
$156-160$ | 3 |
$160-164$ | 1 |
Draw a less than type cumulative frequency curve for this data and use it to compute median height of the children.
Solution To draw the less than type ogive, we plot the points $(124,0),(128,5),(132,13),(136,30)$, $(140,54),(144,70),(148,82),(152,88),(156,92),(160,95),(164,96)$ and join all these point by free hand. Here, $
\dfrac{N}{2}=\dfrac{96}{2}=48
$ We take, $y=48$ in $y$-coordinate and draw a line parallel to $X$-axis, meets the curve at $A$ and draw a perpendicular line from point $A$ to the $X$-axis and this line meets the $X$-axis at the point which is the median i.e., median $=141.17$.Show Answer
Height (in cm)
Number of children
Less than 124
0
Less than 128
5
Less than 132
13
Less than 136
30
Less than 140
54
Less than 144
70
Less than 148
82
Less than 152
88
Less than 156
92
Less than 160
95
Less than 164
96
Size of agricultural holdings (in hec) |
Number of families |
---|---|
$0-5$ | 10 |
$5-10$ | 15 |
$10-15$ | 30 |
$15-20$ | 80 |
$20-25$ | 40 |
$25-30$ | 20 |
$30-35$ | 5 |
Compute median and mode size of the holdings.
Solution (i) Here, $N=200$ Now, $\dfrac{N}{2}=\dfrac{200}{2}=100$, which lies in the interval 15-20. Lower limit, $l=15, h=5, f=80$ and $c f=55$ $
\begin{aligned}
\therefore \quad \text{ Median } = l+\dfrac{\dfrac{N}{2}-c f}{f} \times h=15+\dfrac{100-55}{80} \times 5 \\
& =15+\dfrac{45}{16}=15+2.81=17.81 hec
\end{aligned}
$ (ii) In a given table 80 is the highest frequency. So, the modal class is $15-20$. $
\begin{aligned}
& \text{ Here, } l=15, f_m=80, f_1=30, f_2=40 \text{ and } h=5 \\
& \qquad \begin{aligned}
\text{ Mode } = l+\dfrac{f_m-f_1}{2 f_m-f_1-f_2} \times h \\
& =15+\dfrac{80-30}{2 \times 80-30-40} \times 5 \\
& =15+\dfrac{50}{160-70} \times 5 \\
& =15+\dfrac{50}{90} \times 5=15+\dfrac{25}{9} \\
& =15+2.77=17.77 hec
\end{aligned}
\end{aligned}
$Show Answer
Size of agricultural
holdings (in hec)Number of families $(f_i)$
Cumulative frequency
$0-5$
10
10
$5-10$
15
25
$10-15$
30
55
$15-20$
80
135
$20-25$
40
175
$25-30$
20
195
$30-35$
5
200
Rainfall (in cm) | $0-10$ | $10-20$ | $20-30$ | $30-40$ | $40-50$ | $50-60$ |
---|---|---|---|---|---|---|
Number of days | 22 | 10 | 8 | 15 | 5 | 6 |
Calculate the median rainfall using ogives (or move than type and of less than type)
Solution We observe that, the annual rainfall record of a city less than 0 is 0 . Similarly, less than 10 include the annual rainfall record of a city from 0 as well as the annual rainfall record of a city from 0-10. So, the total annual rainfall record of a city for less than $10 cm$ is $0+22=22$ days. Continuing in this manner, we will get remaining less than 20,30,40,50, and 60 . Also, we observe that annual rainfall record of a city for 66 days is more than or equal to $0 cm$. Since, 22 days lies in the interval $0-10$. So, annual rainfall record for $66-22=44$ days is more than or equal to $10 cm$. Continuing in this manner we will get remaining more than or equal to 20, 30, 40, 50 and 60. Now, we construct a table for less than and more than type. To draw less than type ogive we plot the points $(0,0),(10,22),(20,32),(30,40),(40,55)$, $(50,60),(60,66)$ on the paper and join them by free hand. To draw the more than type ogive we plot the points $(0,66),(10,44),(20,34),(30,26),(40$, $11),(50,6)$ and $(60,0)$ on the graph paper and join them by free hand. $\because$ Total number of days $(n)=66$ Now, $\quad \dfrac{n}{2}=33$ Firstly, we plot a line parallel to $X$-axis at intersection point of both ogives, which further intersect at $(0,33)$ on $Y$-axis. Now, we draw a line perpendicular to $X$-axis at intersection point of both ogives, which further intersect at $(21.25,0)$ on $X$-axis. Which is the required median using ogives. Hence, median rainfall $=21.25 cm$.Show Answer
(i) Less than type
(ii) More than type
Rainfall (in cm)
Number of days
Rainfall (in cm)
Number of days
Less than 0
0
More than or
equal to 066
Less than 10
$0+22=22$
More than or
equal to 10$66-22=44$
Less than 20
$22+10=32$
More than or
equal to 20$44-10=34$
Less than 30
$32+8=40$
More than or
equal to 30$34-8=26$
Less than 40
$40+15=55$
More than or
equal to 40$26-15=11$
Less than 50
$55+5=60$
More than or
equal to 50$11-5=6$
Less than 60
$60+6=66$
More than or
equal to 60$6-6=0$
Duration (in s) | Number of calls |
---|---|
$95-125$ | 14 |
$125-155$ | 22 |
$155-185$ | 28 |
$185-215$ | 21 |
$215-245$ | 15 |
Calculate the average duration (in sec) of a call and also find the median from a cumulative frequency curve.
Solution First, we calculate class marks as follows Here, (assumed mean) $a=170$, and (class width) $h=30$ By step deviation method, $
\text{ Average } \begin{aligned}
(\bar{{}x}) = a+\dfrac{\sum f_i u_i}{\sum f_i} \times h=170+\dfrac{1}{100} \times 30 \\
& =170+0.3=170.3
\end{aligned}
$ Hence, average duration is $170.3 s$. For calculating median from a cumulative frequency curve We prepare less than type or more than type of give We observe that, number of calls in less than $95 s$ is 0 . Similarly, in less than $125 s$ include the number of calls in less than $95 s$ as well as the number of calls from 95-125.s So, the total number of calls less than $125 s$ is $0+14=14$. Continuing in this manner, we will get remaining in less than 155, 185, 215 and $245 s$. Now, we construct a table for less than ogive (cumulative frequency curve). Less than type To draw less than type ogive we plot them the points $(95,0),(125,14)(155,36),(185,64)$, $(215,85),(245,100)$ on the paper and join them by free hand. $
\therefore \quad \dfrac{n}{2}=\dfrac{100}{2}=50
$ Now, point 50 taking on $Y$-axis draw a line parallel to $X$-axis meet at a point $P$ and draw a perpendicular line from $P$ to the $X$-axis, the intersection point of $X$-axis is the median. Hence, required median is 170Show Answer
Duration
(in s)Number of
calls $(f_i)$Class marks
$(x_i)$$\boldsymbol{{}u} _{\boldsymbol{{}i}}=\dfrac{x _{\boldsymbol{{}i}}-\boldsymbol{{}a}}{\boldsymbol{{}h}}$
$\boldsymbol{{}f} _{\boldsymbol{{}i}} \boldsymbol{{}u} _{\boldsymbol{{}i}}$
$95-125$
14
110
-2
-28
$125-155$
22
140
-1
-22
$155-185$
28
$a=170$
0
0
$185-215$
21
200
1
21
$215-245$
15
230
2
30
$\sum f_i=100$
$\sum f_i u_i=1$
Duration (in s)
Number of calls
Less than 95
0
Less than 125
$0+14=14$
Less than 155
$14+22=36$
Less than 185
$36+28=64$
Less than 215
$64+21=85$
Less than 245
$85+15=100$
Distance (in m) | $0-20$ | $20-40$ | $40-60$ | $60-80$ | $80-100$ |
---|---|---|---|---|---|
Number of students | 6 | 11 | 17 | 12 | 4 |
(i) Construct a cumulative frequency table.
(ii) Draw a cumulative frequency curve (less than type) and calculate the median distance drawn by using this curve.
(iii) Calculate the median distance by using the formula for median.
(iv) Are the median distance calculated in (ii) and (iii) same?
Show Answer
Solution
(i)
Distance (in m) |
Number of students $(f_i)$ |
Cumulative frequency $(c f)$ |
---|---|---|
$0-20$ | 6 | 6 |
$20-40$ | 11 | 17 |
$40-60$ | 17 | 34 |
$60-80$ | 12 | 46 |
$80-100$ | 4 | 50 |
(ii)
Distance (in $m$ ) |
Cumulative frequency |
---|---|
0 | 0 |
Less than 20 | 6 |
Less than 40 | 17 |
Less than 60 | 34 |
Less than 80 | 46 |
Less than 100 | 50 |
To draw less than type ogive, we plot the points $(0,0),(20,6),(40,17),(60,34),(80,46)$, $(100,50)$, join all these points by free hand.
Now,
Taking $Y=25$ on $Y$-axis and draw a line parallel to $X$-axis, which meets the curve at point $A$. From point $A$, we draw a line perpendicular to $X$-axis, where this meets that point is the required median i.e., 49.4.
(iii) Now,
$ \dfrac{N}{2}=\dfrac{50}{2}=25 $
which lies is the interval $40-60$.
$ \begin{matrix} \therefore \quad l = 40, h=20, c f=17 \text{ and } f=17 \\ \therefore \quad \text{ Median } = l+\dfrac{\dfrac{N}{2}-c f}{f} \times h \\ & =40+\dfrac{(25-17)}{17} \times 20 \\ & =40+\dfrac{8 \times 20}{17} \\ & =40+9.41 \\ & =49.41 \end{matrix} $
(iv) Yes, median distance calculated by parts (ii) and (iii) are same.